Licchavi Lyceum

ll

Licchavi Lyceum

UPSC Prelims 2011 Solved Paper [Set D]

1. Biodiversity forms the basis for human existence in the following ways :

  1. Soil formation
  2. Prevention of soil erosion
  3. Recycling of waste
  4. Pollination of crops

Select the correct answer using the codes given below :

(a) 1,2,and 3 only

(b) 2,3 and 4 only

(c) 1 and 4 only

(d) 1,2,3 and 4 only

Ans: (d)

Explanation: Biodiversity plays a crucial role in maintaining the ecological balance and providing several benefits to human beings. Out of the given options, all four options are correct.

  1. Soil formation: Biodiversity helps in soil formation by the accumulation of organic matter, which contributes to soil fertility and helps retain soil moisture.
  2. Prevention of soil erosion: The root systems of plants and trees help in holding soil in place, thereby preventing soil erosion.
  3. Recycling of waste: Biodiversity helps in the decomposition of organic waste and recycling of nutrients, which is crucial for the sustenance of life.
  4. Pollination of crops: Many crops depend on pollinators like bees, butterflies, and birds for pollination. Biodiversity loss could have severe impacts on food security and crop production.

2. Aspartame is an artificial sweetener sold in the market. It consist of amino acids and provides calories like other amino acids. Yet, it is used as a low-calorie sweetening agent in food items. What is the basis of this use ?

(a) Aspartame is as sweet as table sugar, but unlike table sugar, it is not readily oxidized in human body due to lack of requisite enzymes.

(b) When aspartame is used in food processing, the sweet taste remains, but it becomes resistant to oxidation.

(c) Aspartame is as sweet as sugar, but after ingestion into the body, it is converted into metabolites that yield no calories.

(d) Aspartame is several times sweeter than table sugar, hence food items made with small quantities of aspartame yield fewer calories on oxidation.

Ans: (d)

Solution: Aspartame is an artificial, low-calorie sweetener that is used as a sugar substitute in a variety of food and beverage products. It is made up of two amino acids, aspartic acid and phenylalanine, and is approximately 200 times sweeter than sugar.

Aspartame does not oxidize in human body.

3.What was the purpose with which sir William wedderburn and W.S.Caine had set up the Indian parliamentary committee in 1893 ?

(a)To agitate for Indian political reforms in the house of commons.

(b)To campaign for the entry of Indians into the imperial judiciary.

(c)To facilitate a discussion on India’s independence in the British parliament.

(d)To agitate for the entry of eminent Indians into the British parliament.

Ans: (a)

Solution: The Indian Parliamentary Committee was set up by Sir William Wedderburn and W.S. Caine in 1893 with the aim of advocating for Indian political reforms in the House of Commons in the UK.

The committee aimed to promote Indian interests in the British Parliament and to highlight issues such as poverty, land tenure, and civil rights in India. The committee also worked to raise awareness of the need for Indian representation in the legislative councils in India. The Indian Parliamentary Committee played a significant role in promoting Indian political interests in the UK and was an important precursor to later Indian nationalist movements. Therefore, option (a) is the correct answer.

4.What is the difference between a CFL and an LED lamp ?

(a) To produce light, a CFL uses mercury vapour and phosphor while an LED lamp uses semi-conductor material.

(b) The average life span of a CFL is much longer than that of an LED lamp.

(c) A CFL is less energy-efficient as compared to an LED lamp.

Which of the statements given above is/are correct ?

(a)1 only

(b)2 and 3 only

(c) 1 and 3 only

(d)1,2,and 3

Ans: (c)

Solution: 

Parameter CFL LED
Energy Efficiency Less efficient, consumes more power Highly efficient, consumes less power
Lifespan Average lifespan of 10,000 hours Long lifespan of 25,000 to 50,000 hours
Start-up Time Takes a few seconds to reach full brightness Instantly turns on
Heat Emission Produces more heat, can be a fire hazard Produces less heat, cool to the touch
Light Quality Harsh, flickers at low frequency High-quality, natural-looking light
Environmental Impact Contains mercury, not easily recyclable Environmentally friendly, recyclable
Cost Cheaper than LED, but more expensive than bulb More expensive upfront, but cheaper later
How CFL produce light?

CFLs (Compact Fluorescent Lamps) produce light through a process called fluorescence. Inside a CFL bulb, there is a small amount of mercury vapor and a phosphor coating on the inner surface of the glass tube.
When electricity flows through the tube, it ionizes the gas and creates ultraviolet (UV) light. The UV light strikes the phosphor coating, causing it to fluoresce or emit visible light. The phosphor coating is designed to emit white light, which gives the CFL bulb its characteristic color temperature.

CFLs also contain a small electronic ballast that regulates the flow of current through the tube. This ballast is responsible for starting and maintaining the light output of the CFL bulb.

5. Recently , “oil zapper’’ was in the news. What is it ?

(a) It is an eco-friendly technology for the remediation of oily sludge and oil spills.

(b) It is the latest technology developed for under-sea oil exploration.

(c) It is a genetically engineered high biofuel-yielding maize variety.

(d) It is the latest technology to control the accidentally caused flames from oil wells.

Answer: (a)

Solution: The “oil zapper” is an eco-friendly technology used for the remediation of oily sludge and oil spills. It is a microbial consortium developed by the National Environmental Engineering Research Institute (NEERI), Nagpur, India. The technology is based on the principle of bioremediation, which involves the use of microorganisms to degrade or break down pollutants into harmless substances.

The oil zapper contains a mixture of naturally occurring microorganisms that are capable of degrading a wide range of hydrocarbons present in crude oil and other petroleum products. The microorganisms in the oil zapper break down the hydrocarbons into carbon dioxide and water, which are harmless to the environment.

The use of oil zapper technology is considered to be an effective and environmentally friendly way of cleaning up oil spills and other forms of hydrocarbon contamination. Therefore, option (a) is the correct answer.

6. A married couple adopted a male child. A few years later, twin boys were born to them. The blood group of the couple is AB positive and O negative. The blood group of the three sons is A positive, B positive , and O positive. The blood group of the adopted son is ?

(a) O positive

(b) A positive

(c) B positive

(d) Cannot be determined on the basis of the given data.

Ans: (a)

Solution:

How the blood group of the child is decided?

The blood group of a child is determined by the inheritance of genes from the parents. Specifically, blood groups are determined by the presence or absence of certain proteins, called antigens, on the surface of red blood cells. The most important blood group systems are ABO and Rh.

The ABO blood group system is determined by the presence or absence of two antigens, A and B. There are four possible blood types in the ABO system: A, B, AB, and O. A person inherits one ABO gene from each parent, and the combination of these genes determines their blood type.

The Rh blood group system is determined by the presence or absence of the Rh factor antigen. A person who has the Rh antigen is Rh-positive, while a person who does not have the antigen is Rh-negative. The Rh factor is inherited independently of the ABO system.

When a child is conceived, it inherits one ABO gene and one Rh gene from each parent. The possible combinations of these genes determine the child’s blood group. For example, if both parents are ABO blood type A and Rh-positive, their child could be A-positive, A-negative, O-positive, or O-negative, depending on the combination of genes inherited from the parents.

7. Mahatma Gandhi said that some of his deepest convictions were reflected in a book titled, “unto this last’’ and the book transformed his life. What was the message from the book that transformed mahatma Gandhi ?

(a)Uplifting the oppressed and poor is the moral responsibility of an educated man

(b)The good of individual is contained in the good of all.

(c)The life of celibacy and spiritual pursuit are essential for a noble life.

(d)All the statements (a), (b) and (c) are correct in this context.

Ans: (b)

Solution: This quote by Mahatma Gandhi suggests that individual well-being is interconnected with the well-being of society as a whole. In other words, when the community is thriving and prosperous, individuals within that community are more likely to experience happiness and fulfillment.

8. With reference to Indian freedom struggle, Usha Mehta is well-known for ?

(a)Running the secret congress radio in the wake of quit India movement.

(b)Participating in the second round table conference.

(c)Leading a contingent of Indian national army.

(d)Assisting in the formation of Interim government under Pandit Jawaharlal Nehru.

Ans: (a)

Solution: Usha Mehta was a prominent figure in the Indian freedom struggle, and she is best known for running the secret Congress Radio during the Quit India Movement in 1942. She set up an underground radio station in Mumbai, which broadcasted news about the freedom struggle and speeches by national leaders, including Mahatma Gandhi.

The Congress Radio played a crucial role in mobilizing public support for the Quit India Movement and keeping people informed about the latest developments in the freedom struggle. Usha Mehta and her team risked their lives to keep the radio station running and to disseminate information to the people.

Usha Mehta was also a close associate of Mahatma Gandhi, and she actively participated in the freedom struggle through various means. After Independence, she remained active in social and political causes, and worked towards promoting education and women’s empowerment.

9. A new optical disc format known as the blu-ray disc (BD) is becoming popular. In what way is it different from the traditional DVD ?

1.DVD supports standard definition video while BD supports high definition video.

2.Compared to a DVD, the BD format has several times more storage capacity.

3.Thickness of BD is 2.4 mm while that of DVD is 1.2 mm.

Which of the statements given above is/are correct ?

(a)1 only

(b)1 and 2 only

(c)2 and 3 only

(d)1,2 and 3

Ans: (b)

Solution: Blu-ray Disc (BD) and DVD are both optical disc formats used for storing digital data. However, there are some significant differences between the two:

  1. Blu-ray Discs support high definition (HD) video, while DVDs support standard definition (SD) video. This means that Blu-ray Discs offer better picture quality than DVDs.
  2. Blu-ray Discs have a larger storage capacity compared to DVDs. A single-layer Blu-ray Disc can hold up to 25 GB of data, while a dual-layer disc can hold up to 50 GB of data. In contrast, a DVD can hold up to 4.7 GB of data for a single-layer disc, and up to 8.5 GB of data for a dual-layer disc.
  3. The thickness of a Blu-ray Disc is 2.4 mm, while the thickness of a DVD is 1.2 mm. However, this difference in thickness is not considered significant in terms of functionality.

Therefore, option (b) 1 and 2 only is the correct answer.

10. With reference to the period of Indian freedom struggle, which of the following was/were recommended by the Nehru report ?

1.Complete independence for India.

2.Joint electorates  for reservation of seats for minorities.

3.Provision of fundamental rights for the people of India in the constitution.

Select the correct answer using the codes given below :

(a)1 only

(b)2 and 3 only

(c)1 and 3 only

(d)1,2 and 3

Ans: (b)

Solution: The Nehru Report was a significant document produced by the Indian National Congress in 1928, which proposed a constitution for India. It included the following recommendations:

  1. The report demanded dominion status and not the complete independence for India.
  2. The report recommended that the constitution should include a bill of rights to protect the fundamental rights of the people of India. This was a significant proposal, as it laid the foundation for the inclusion of fundamental rights in the Indian Constitution.
  3. The Nehru Report recommended joint electorates for all, without any reservation of seats for minorities. However, this recommendation was opposed by some Muslim leaders, who argued for separate electorates for Muslims.

11. Among the following states, which one has the most suitable climatic conditions for the cultivation of a large variety of orchids with minimum cost of production, and can develop an export oriented industry in this field ?

(a)Andhra Pradesh

(b)Arunachal Pradesh

(c)Madhya Pradesh

(d)Uttar Pradesh

Ans: (b)

Solution: Among the given options, Arunachal Pradesh has the most suitable climatic conditions for the cultivation of a large variety of orchids with minimum cost of production. Arunachal Pradesh is known as the “Orchid State of India” due to its vast range of orchids.

The state’s climate is ideal for orchid cultivation as it has a tropical to subtropical climate, high rainfall, and a wide range of altitudes. Arunachal Pradesh has the potential to develop an export-oriented industry in the orchid cultivation field due to its diverse orchid species, low production costs, and high demand in the international market.

12. Which one of the following  is not a site for in-situ method of conservation of flora ?

(a)Biosphere reserve

(b)Botanical garden

(c)National park

(d)Wildlife sanctuary

Ans: (b)

Solution: Botanical garden is not a site for in-situ method of conservation of flora.

In-situ conservation refers to the conservation of species in their natural habitats, which includes protected areas such as national parks, wildlife sanctuaries, and biosphere reserves. These areas are designated for the preservation of ecosystems and their associated flora and fauna.

Botanical gardens, on the other hand, are ex-situ conservation sites where plants are conserved outside their natural habitats. They are usually established for research, education, and public display purposes. While botanical gardens may have conservation objectives, they do not involve the preservation of plant species in their natural habitats and are therefore not considered in-situ conservation sites.

13. Consider the following statements :

In India, a metropolitan planning committee :

1. Is constituted under the provisions of the constitution of India.

2. Prepares the draft development plans for metropolitan area.

3. Has the sole responsibility for implementing government sponsored schemes in the metropolitan area.

Which of the statements given above is/are correct ?

(a) 1 and 2 only

(b) 2 only

(c) 1,3 only

(d) 1,2 and 3

Ans: (a)

Solution: Statement 1 is correct. The Metropolitan Planning Committee (MPC) is constituted under the provisions of the Constitution of India. Article 243 of the Constitution provides for the constitution of an MPC for every metropolitan area.

Statement 2 is also correct. The MPC is responsible for preparing a draft development plan for the metropolitan area based on the spatial development strategy and plans prepared by the state government and the local planning authorities.

Statement 3 is incorrect. The MPC does not have the sole responsibility for implementing government-sponsored schemes in the metropolitan area. The implementation of schemes is the responsibility of the concerned local bodies and state government departments. The MPC is only responsible for ensuring that the plans and schemes prepared by the local bodies and state government departments are in conformity with the development plan prepared by the MPC.

14. What is the difference between “vote-on-account” and interim budget ?

  1. The provision of a “vote-on-account’’ is used by a regular government, while an ”interim budget’’ is a provision used by a caretaker government.
  2. A “vote-on-account’’ only deals with the expenditure in government is budget, while an “interim budget’’ includes both expenditure and receipts.

Which of the statements given above is/are correct ?

(a)1 only

(b)2 only

(c)Both 1 and 2

(d)Neither 1 nor 2

Ans: (b)

Explanation: A ‘vote-on-account’ and an ‘interim budget’ are two different financial provisions used by the government in India.

A vote-on-account is a provision that allows the government to obtain the vote of Parliament for a sum of money to meet the expenditure of the government for a part of the financial year, usually the first quarter. The purpose of the vote-on-account is to provide essential funds to the government to carry out its functions until the full budget is passed. It only deals with expenditure.

On the other hand, an interim budget is a statement of accounts that presents the financial position of the government for the current financial year, as well as the estimates of expenditure and receipts for the upcoming financial year. It is presented by a government that is going out of power before the end of its term, as well as by a new government that has taken charge and needs time to prepare a full budget. It includes both expenditure and receipts.

15. Regarding the international monetary fund, which one of the following statements is correct ?

(a) It can grant to any country.

(b) It can grant loans to only developed countries.

(c) It grants loans to only member countries.

(d) It can grant loans to the central bank of a country.

Ans: (c)

Solution: The International Monetary Fund (IMF) is an international organization that provides financial assistance and advice to its member countries. It was established in 1944 to promote international monetary cooperation, exchange stability, and orderly exchange arrangements.

The IMF grants loans to member countries that are facing balance of payments problems or other economic difficulties. The loans come with conditions, such as implementing economic reforms and policies that are intended to address the root causes of the country’s economic problems.

The IMF does not grant loans to non-member countries or to the central bank of a country. Additionally, the IMF is not limited to providing loans to developed countries only. It provides assistance to all member countries regardless of their level of development.

16. In the union budget 2011-12, a full exemption from the basic customs duty was extended to the bio-based asphalt (Biosphalt). What is the importance of this material ?

1.    Unlike traditional  asphalt, bio-asphalt is not based on fossil fuels.

2.    Biosphalt can be made from non-renewable resources.

3.    Bioasphalt can be made from organic waste materials.

4.    It is eco-friendly to use bioasphalt for surfacing of the roads.

Which the correct answer using the codes given below ?

(a)  1,2,and 3 only

(b) 1,3 and 4 only

(c) 2 and 4 only

(d) 1,2,3, and 4

Ans: (b)

17.  Consider the following:

  1. Carbon dioxide
  2. Oxides of nitrogen
  3. Oxides of Sulphur

Which of the above is/are the emission/emissions from coal combustion at thermal power plants?

(a) 1 only

(b) 2 and 3 only

(c) 1 and 3 only

(d) 1,2, and 3

Ans: (d)

Solution: All of the above emissions, i.e., Carbon dioxide, Oxides of nitrogen, and Oxides of Sulphur, are the emissions from coal combustion at thermal power plants.

Burning coal releases carbon dioxide into the atmosphere, which is a major contributor to global warming. Coal also contains sulfur, which, when burned, produces sulfur dioxide (SO2) and other sulfur compounds that can react with air and water to form sulfuric acid (H2SO4), contributing to acid rain. Nitrogen is also present in coal, and when burned, it reacts with oxygen to form nitrogen oxides (NOx), which contribute to smog and acid rain.

18. Satellites used for telecommunication relay are kept in a geostationary orbit. A satellite is said to be in such as orbit when :

1. The orbit is geosynchronous

2. The orbit is circular.

3. The orbit lies in the plane of the earth’s equator.

4. The orbit is at an altitude of 22,236.

Which the correct answer using the codes given below :

(a) 1,2,and 3 only

(b) 1,3 and 4 only

(c) 2 and 4 only

(d) 1,2,3, and 4

Ans: (A)

Solution : Please note that a Geostationary orbit is a geosynchronous orbit that is circular and at zero inclination, that is, directly above the equator. The height is 22236 miles BUT NOT in Kilometers, in Kilometers it is 35,786.

Location of Geostationary Satellite
Location of Geostationary Satellite

19. India has experienced persistent and high food inflation in the recent past. What could be the reasons?

  1. Due to a gradual switchover to the cultivation of commercial crops, the area under the cultivation of food grains has steadily decreased in the last five years by about 30%.
  2. As a consequence of increasing incomes, the consumption patterns of the people have undergone a significant change.
  3. The food supply chain has structural constraints.

Which of the statements given above are correct ?

(a) 1 and 2 only

(b) 2 and 3 only

(c) 1 and 3 only

(d) 1,2, and 3

Ans: (b)

Solution: According to RBI, trend of food inflation was pointing at not only structural demand-supply mismatches in commodities comprises the essential consumption basket but also at changing consumption patterns.

20. At present , scientists can determine the arrangement or relative positions of genes or DNA sequences on a chromosome. How does this knowledge benefit us?

  1. It is possible to know the pedigree of livestock.
  2. It is possible to understand the causes of all human diseases.
  3. It is possible to develop disease-resistant animal breeds.

Which of the statements give above is/are correct ?

(a) 1 and 2 only

(b) 2 only

(c) 1 and 3 only

(d) 1,2, and 3

Ans: (c)

Solution:

The correct statement is (c) 1 and 3 only.

Knowing the arrangement or relative positions of genes or DNA sequences on a chromosome can benefit us in several ways. Two of them are:

  1. It is possible to know the pedigree of livestock: By understanding the genetic makeup of animals, scientists can determine which animals are more likely to exhibit desirable traits, such as disease resistance, high productivity, or superior meat quality. This can help breeders produce healthier and more productive animals, which can ultimately benefit farmers and consumers.
  2. It is possible to develop disease-resistant animal breeds: By identifying the genes responsible for disease resistance, scientists can breed animals that are less susceptible to certain diseases. This can reduce the need for antibiotics and other medications, which can help prevent the spread of antibiotic-resistant bacteria and improve animal welfare.

However, it is not correct to say that knowing the arrangement of genes on a chromosome can help us understand the causes of all human diseases (statement 2). While understanding the genetic basis of many diseases can be very helpful, there are many factors that contribute to disease development, including environmental factors, lifestyle choices, and complex interactions between multiple genes. Therefore, it is not accurate to say that we can understand the causes of all human diseases just by knowing the arrangement of genes on a chromosome.

21. In terms of economy, the visit by foreign nationals to witness the XIX common wealth games in India amounted to ?

(a) Export

(b) Import

(c) Production

(d) Consumption

Ans: (a)

Solution: The visit by foreign nationals to witness the XIX Commonwealth Games in India would be considered as an “Export” in terms of the economy. This is because the money spent by foreign visitors is considered an inflow of foreign currency into the country, which is beneficial for the economy. Additionally, it can also contribute to the growth of various industries such as tourism, hospitality, transportation, and retail, generating employment opportunities and generating revenue for the country.

22. Microbial fuel cells are considered a source of sustainable energy. Why ?

1. They use living organisms as catalysts to generate electricity from certain substrates.

2. They use a variety of inorganic materials as substrates.

3. They can be installed in waste water treatment plants to cleanse water and produce electricity.

Which of the following statements given above is/ are correct ?

(a) 1 only

(b) 2 and 3 only

(c) 1 and 3 only

(d) 1,2,and 3

Ans: (c)

Solution: Microbial fuel cells (MFCs) are devices that use microorganisms to convert organic matter into electricity. The microorganisms used in MFCs act as catalysts to generate electricity from certain substrates. Therefore, statement 1 is correct.

In terms of substrates, MFCs can use a variety of organic materials, including waste water, sewage sludge, and agricultural waste. However, they do not use inorganic materials as substrates. Therefore, statement 2 is incorrect.

One of the most promising applications of MFCs is in wastewater treatment plants. MFCs can be installed in wastewater treatment plants to cleanse water and simultaneously produce electricity. Therefore, statement 3 is correct.

Hence, the correct statements are 1 and 3 only.

23. Which one of the following statements appropriately describes the “fiscal stimulus” ?

(a) It is a massive investment by the government in manufacturing sector to ensure the supply of goods to meet the demand surge caused by rapid economic growth.

(b) It is an intense affirmative action of the government to boost economic activity in the country.

(c) It is government’s intensive action on financial institutions to ensure disbursement of loans to agriculture and allied sectors to promote greater food production and contain food inflation.

(d) It is an extreme affirmative action by the government to pursue its policy of financial inclusion

Ans: (b)

Solution: The statement that appropriately describes “fiscal stimulus” is (b) It is an intense affirmative action of the government to boost economic activity in the country.

Fiscal stimulus refers to the use of government spending and tax policies to boost economic activity during an economic downturn. When the economy is in a recession or a slowdown, the government may use fiscal policy tools to increase government spending and/or cut taxes to stimulate aggregate demand and boost economic growth.

Option (a) is incorrect as it describes a government investment in the manufacturing sector to meet the demand surge caused by rapid economic growth, whereas fiscal stimulus is typically used during an economic downturn.

Option (c) is incorrect as it describes the government’s intensive action on financial institutions to ensure disbursement of loans to agriculture and allied sectors to promote greater food production and contain food inflation, which is not directly related to fiscal stimulus.

Option (d) is incorrect as it describes an extreme affirmative action by the government to pursue its policy of financial inclusion, which is not related to fiscal stimulus.

Therefore, option (b) is the correct statement that describes fiscal stimulus.

24. The formation of ozone hole in the Antarctic region has been a cause of concern. What could be the reason for the formation of this hole?

(a) Presence of prominent tropospheric turbulence and inflow of chlorofluorocarbons.

(b) Presence of prominent polar front and stratospheric clouds and inflow of chlorofluorocarbons.

(c) Absence of polar front and stratospheric clouds; and inflow of methane and chlorofluorocarbons.

(d) Increased temperature at polar region due to global warming.

Ans: (b)

Solution: The reason for the formation of the ozone hole in the Antarctic region is (b) Presence of prominent polar front and stratospheric clouds; and inflow of chlorofluorocarbons.

The ozone layer is a layer in the Earth’s atmosphere that absorbs most of the Sun’s ultraviolet (UV) radiation. Ozone depletion occurs when the ozone layer is damaged by certain chemicals, such as chlorofluorocarbons (CFCs) and other ozone-depleting substances.

In the Antarctic region, the formation of the ozone hole is mainly attributed to the presence of prominent polar front and stratospheric clouds, which leads to the formation of the polar stratospheric clouds (PSCs). These clouds provide a surface for chemical reactions to occur that lead to the depletion of ozone. Additionally, the inflow of chlorofluorocarbons and other ozone-depleting substances into the atmosphere contributes to the depletion of the ozone layer.

Option (a) is incorrect as prominent tropospheric turbulence is not a significant factor in the formation of the ozone hole, and the inflow of chlorofluorocarbons is the primary reason for ozone depletion, not tropospheric turbulence.

Option (c) is incorrect as methane does not contribute significantly to ozone depletion, and the absence of polar front and stratospheric clouds would not lead to the formation of the polar stratospheric clouds that are responsible for the chemical reactions that deplete ozone.

Option (d) is incorrect as the increased temperature at the polar region due to global warming is not directly related to the formation of the ozone hole.

Therefore, the correct option is (b).

25. Consider the following actions which the government can take :

1. Devaluing the domestic currency

2. Reduction in the export subsidy

3. Adopting suitable policies which attract greater FDI and more funds from FIIs

Which of the above action/actions can help in reducing the current account deficit ?

(a) 1 and 2

(b) 2 and 3

(c) 3 only

(d) 1 and 3

Ans: (d)

Solution: The action/actions that can help in reducing the current account deficit are (d) 1 and 3.

The current account deficit is the difference between a country’s total imports of goods, services, and transfers and its total exports of goods, services, and transfers. A country can reduce its current account deficit by increasing exports or decreasing imports.

S1: Devaluing the domestic currency can make the country’s exports cheaper and more competitive in the international market, which can lead to an increase in exports and a decrease in imports, thereby reducing the current account deficit. This strategy has been widely used by China.

S2: Reducing export subsidy can lead to a decrease in exports, which can further widen the current account deficit. Therefore, option 2 is not a suitable action for reducing the current account deficit.

S3: Attracting greater Foreign Direct Investment (FDI) and more funds from Foreign Institutional Investors (FIIs) (option 3) can help in financing the current account deficit. FDI and FII inflows can provide the required foreign exchange to pay for imports and help reduce the current account deficit.

Therefore, the correct option is (d) 1 and 3.

26. The constitution (seventy-third amendment) act, 1992, which aims at promoting the panchayati raj institutions in the country, provides for which of the following ?

1. Constitution of district planning committees

2. State election commissions to conduct all panchayat elections

3. Establishment of state finance commission

State the correct answer using the codes given below :

(a) 1 only

(b) 2 and 3 only

(c) 2 and 3 only

(d) 1,2,and 3

Ans: (d)

Solution: The Constitution (Seventy-third Amendment) Act, 1992, which aims at promoting the Panchayati Raj institutions in the country, provides for the following:

Constitution of District Planning Committees (DPCs) (option 1) to prepare the development plans for the district and consolidate the plans prepared by the Panchayats and Municipalities in the district.

State Election Commissions (SECs) to conduct all Panchayat elections (option 2).

Establishment of State Finance Commissions (SFCs) (option 3) to review the financial position of the Panchayats and make recommendations to the Governor on the principles that should govern:
(i) the distribution between the State and the Panchayats of the net proceeds of taxes, duties, tolls, and fees leviable by the State, which may be divided between them under the State law;

(ii) the determination of the taxes, duties, tolls, and fees which may be assigned to or appropriated by the Panchayats; the grants-in-aid to the Panchayats from the Consolidated Fund of the State; and the measures needed to improve the financial position of the Panchayats.

Therefore, the correct answer is (d) 1, 2, and 3.

27. Two important rivers- one with its source in Jharkhand (and known by a different name in odisha), and another, with its source in odisha- merge at a place only a short distance from the coast of bay of Bengal before flowing into the sea. This is an important site of wildlife and biodiversity and a protected area. Which one of the following could be this ?

(a) Bhitarkanika

(b) Chandipur

(c) Gopalpur

(d) Simlipal

Ans: (a)

Solution: The description provided in the question refers to the merging of two rivers with different names in Jharkhand and Odisha and then flowing into the Bay of Bengal. The protected area with rich biodiversity and wildlife could refer to the Bhitarkanika National Park and Wildlife Sanctuary in Odisha.

Bhitarkanika National Park

28. A rapid increase in the rate of inflation is sometimes attributed to the “base effect”. What is “base effect” ?

(a) It is the impact of drastic deficiency in supply due to failure of crops.

(b) It is the impact of the surge in demand due to rapid economic growth.

(c) It is the impact of the price levels of previous year on the calculation of inflation rate.

(d) None of the statements (a), (b) and (c) given above is correct in this context.

Ans: (c)

Solution: The base effect refers to the impact of the price levels of the previous year on the calculation of inflation rate in the current year. Inflation is measured as the percentage change in the price level of a basket of goods and services over time. When there is a sudden increase in the inflation rate, it may be due to the base effect, which occurs when the current inflation rate is compared to a low inflation rate in the previous year.

This comparison can make the current inflation rate appear higher than it actually is, as it is measured relative to a lower base. Conversely, a low inflation rate in the current year compared to a high inflation rate in the previous year may make the current rate appear lower than it actually is, due to the same effect.

29. India is regarded as a country with “Demographic Dividend’’. This is due to ?
(a) Its high population in the age group below 15 years.

(b) Its high population in the age group of 15-64 years.

(c) Its high population in the age group above 65 years.

(d) Its high total population.

Ans: (b)

Solution: India is regarded as a country with “Demographic Dividend” due to its high proportion of the working-age population (15-64 years) relative to the dependent population (those under 15 years and above 65 years). This demographic profile has the potential to provide a boost to economic growth and development as a large and productive workforce can contribute to the economy through increased consumption, investment, and innovation.

However, realizing the full potential of the demographic dividend requires investments in education, health, and skills training to ensure that the workforce is equipped with the necessary skills to participate fully in the economy.

30. Regarding “carbon credits’’ , which one of the following statements is not correct ?

(a) The carbon credit system was ratified in conjunction with the Kyoto protocol.

(b) Carbon credits are awarded to countries or groups that have reduced greenhouse gases below their emission quota.

(c) The goal of the carbon credit system is to limit the increase of carbon dioxide emission.

(d) Carbon credits are traded at a price fixed from time to time by the united nations environment programs.

Ans: (d)

Solution: Carbon credits are not traded at a price fixed from time to time by the United Nations Environment Programs (UNEP). Instead, the market price of carbon credits is determined by supply and demand in various carbon markets, such as the European Union Emissions Trading System (EU ETS) and the Clean Development Mechanism (CDM) market.

The carbon credit system was ratified in conjunction with the Kyoto protocol, which is an international agreement aimed at reducing greenhouse gas emissions. Carbon credits are awarded to countries or groups that have reduced greenhouse gases below their emission quota.
The goal of the carbon credit system is to limit the increase of carbon dioxide emissions.

31. Consider the following :

1. Right to education

2. Right to equal access to public service

3. Right to food

Which of the above is/are human right/human rights under “universal declaration of human rights’’ ?

(a) 1 only

(b) 1 and 2 only

(c) 3 only

(d) 1,2,and 3

Ans: (d)

Solution: All three of the above are human rights under the “Universal Declaration of Human Rights.” Therefore, the correct answer is (d) 1, 2, and 3.

The right to education is recognized as a fundamental human right under Article 26 of the Universal Declaration of Human Rights. It states that everyone has the right to education, and that education shall be free, at least in the elementary and fundamental stages.

The right to equal access to public services is also recognized as a fundamental human right under Article 21 of the Universal Declaration of Human Rights. It states that everyone has the right to take part in the government of their country, directly or through freely chosen representatives, and that everyone has the right of equal access to public service in their country.

The right to food is recognized as a basic human right under Article 25 of the Universal Declaration of Human Rights. It states that everyone has the right to a standard of living adequate for the health and well-being of themselves and their family, including food, clothing, housing, and medical care.

32. There is a concern over the increase in harmful algal blooms in the seawater of India. What could be the causative factors for this phenomenon ?

1. Discharge of nutrients from the estuaries

2. Run-off from the land during the monsoon

3. Upwelling in the seas

Select the correct answer from the codes given below :

(a) 1 only

(b) 1 and 2 only

(c) 2 and 3 only

(d) 1,2,and 3

Ans: (d)

Solution: Harmful algal blooms are caused by the rapid growth and accumulation of certain species of algae in marine or freshwater environments, which can produce harmful toxins that can affect the health of humans and wildlife.

The causative factors for the increase in HABs in the seawater of India are likely to be the discharge of nutrients from the estuaries and the run-off from the land during the monsoon, both of which can increase the nutrient levels in the water and promote the growth of harmful algae.

However, the upwelling in the seas, which brings nutrient-rich water to the surface, can also contribute to it.

33. Consider the following :

1. Photosynthesis

2. Respiration

3. Decay of organic matter

4. Volcanic action

Which of the above add carbon dioxide to the carbon cycle on earth ?

(a) 1 and 4 only

(b) 2 and 3 only

(c) 2, 3 and 4 only

(d) 1,2,3 and 4

Ans: (c)

Solution: Photosynthesis and respiration are two opposite processes that balance each other in terms of carbon dioxide exchange. During photosynthesis, plants absorb carbon dioxide and release oxygen, while during respiration, they absorb oxygen and release carbon dioxide.

Respiration, Decay of organic matter and volcanic action also release carbon dioxide into the atmosphere.

During photosynthesis the oxygen is released.

34. Recently, the USA decided to support India’s membership in multi-lateral export control regimes called the “Australia group” and the “Wassenaar arrangement. What is the difference between them ?

1. The Australia group is an informal arrangement which aims to allow exporting countries to minimize the risk of assisting chemical and biological weapons proliferation, whereas the Wassenaar arrangement is a formal group under the OECD holding identical objectives.

2. The Australia group comprises predominantly of Asian, African and north American countries, whereas the member countries of Wassenaar arrangement are pre-dominantly from the European union and American continents.

Which of the statements given above is/are correct ?

(a) 1 only

(b) 2 only

(c) Both 1 and 2

(d) Neither 1 nor 2

Ans: (D)

Solution: The Australia group is an informal arrangement which aims to allow exporting countries to minimize the risk of assisting chemical and biological weapons proliferation, whereas the Wassenaar arrangement is a formal group under the OECD holding identical objectives.

The Australia group is an informal forum of countries that aims to prevent the spread of chemical and biological weapons by controlling the export of related materials and technologies. The group was established in 1985 and consists of 43 members, including the United States, European Union, and Japan.

The Wassenaar Arrangement, on the other hand, is a formal multilateral export control regime that regulates the trade of conventional weapons and dual-use goods and technologies. It was established in 1996 and consists of 42 member countries, including the United States, European Union, and Canada.

35. The surface of a lake is frozen in severe winter, but the water at its bottom is still liquid. What is the reason?

(a) Ice is a bad conductor of heat.

(b) Since the surface of the lake is at the same temperature as the air, no heat is lost.

(c) The density of water is maximum at 4°c.

(d) None of the statements (a), (b) and (c) given is correct.

Ans: (c)

Solution: The reason the surface of a lake is frozen in severe winter while the water at its bottom is still liquid is due to the fact that ice is a bad conductor of heat. This means that heat is not easily transferred from the warmer water at the bottom of the lake to the colder air above.

Additionally, the surface of the lake may also lose heat through radiation to the cold sky, further reducing the temperature of the surface water. As a result, the water at the surface freezes while the water at the bottom, which is insulated by the ice layer above, remains liquid. Therefore, option (a) should be the correct answer, but as per UPSC Official Answer Key the Answer is (c).

36. A sandy and saline area is the natural habitat of an Indian animal species. The animal has no predators in that area but its existence is threatened due to the destruction of its habitat. Which one of the following could be that animal ?

(a) Indian wild buffalo

(b) Indian wild ass

(c) Indian wild boar

(d) Indian gazelle

Ans: (b)

Solution: Indian Wild Ass is found in the dry and arid regions of India, specifically in the Rann of Kutch in Gujarat, which is the largest salt desert in the world. The Rann of Kutch is a harsh and unforgiving environment, with temperatures that can reach up to 120°F and very little water. Despite these harsh conditions, the it has adapted to thrive in this unique ecosystem.

37. La Nina is suspected to have caused recent floods in Australia. How is La Nina different from EI Nino ?
1. La Nina is characterized by un-usually cold ocean temperature in equatorial Indian ocean whereas EI Nino is characterized by unusually warm ocean temperature in the equatorial pacific ocean.
2. EI Nino has adverse effect on south-west monsoon of India, but La Nina has no effect on monsoon climate.
Which of the statements given above is/are correct ?

(a) 1 only

(b) 2 only

(c) Both 1 and 2

(d) Neither 1 nor 2

Ans: (d)

Solution:

S1: La Nina and El Nino are opposite phases of a natural climate pattern called the El Nino-Southern Oscillation (ENSO). While El Nino is characterized by unusually warm ocean temperatures in the equatorial Pacific Ocean, La Nina is characterized by unusually cold ocean temperatures in the same region. Hence S1 is incorrect. 

S2: La Nina and El Nino can both have significant impacts on global weather patterns, including the amount and distribution of rainfall. However, the statement that La Nina has no effect on the monsoon climate is incorrect, as La Nina has been known to cause droughts in some parts of the world, including Australia.

38. The tendency for increased litigation was visible after the introduction of the land settlement system of lord Cornwallis in 1793. The reason for this is normally traced to which of the following provisions ?

(a) Making zamindar’s position stronger vis-à-vis the ryot.

(b) Making east India company an overlord of zamindars.

(c) Making judicial system more efficient.

(d) None of the (a), (b) and (c) above.

Ans: (D)

Solution: The increased rate of litigation was the establishment of transparent judiciary and introduction of Indian Panel Code by Lord Cornwallis. 

39. Which one of the following observation is not true about the quit India movement of 1942 ?

(a) It was a non-violent movement.

(b) It was led by mahatma Gandhi.

(c) It was a spontaneous movements.

(d) It did not attract the labour class in general.

Ans: (b)

Solution: The Quit India Movement was started by Mahatma Gandhi, who was one of the most prominent leaders of the Indian independence movement. Gandhi called for the Quit India Movement as a mass civil disobedience campaign against British colonial rule in India. The movement aimed to bring about the withdrawal of British colonial power from India and achieve independence for the country. However, important leaders including Gandhi was in jail during the movement and the movement is considered to be a leaderless movement. 

40. Which amongst the following provided a common factor for tribal insurrection in India in the 19th century ?

(a) Introduction of a new system of land revenue and taxation of tribal products.

(b) Influence of foreign religious missionaries in tribal areas.

(c) Rise of a large number of money lenders, traders and revenue farmers as middlemen in tribal areas.

(d) The complete disruption of the old agrarian order of the tribal communities.

Ans: (d)

Solution: The options are very close. The best option will be (d) as the activities caused complete disruption of agriculture practices of tribal communities.

41. India maintained its early cultural contacts and trade links with southeast Asia across the bay of Bengal. For this pre-eminence of early maritime history of bay of Bengal , which of the following could be the most convincing explanation/explanations ?

(a) As compared to other countries, India had a better ship-building technology in ancient and medieval times.

(b) The rulers of southern India always patronized traders , Brahmin priests and Buddhist monks in this context.

(c) Monsoon winds across the bay of Bengal facilitated sea voyages.

(d) Both (a) and (b) are convincing explanations in this context.

Ans: (c)

Solution: The monsoon winds across the Bay of Bengal facilitated sea voyages, making it easier for Indian traders and merchants to sail across the sea and establish trade links with Southeast Asia. The predictable nature of the monsoon winds allowed Indian traders to plan their voyages with greater certainty and safety.

42. What is the difference between blue-tooth and Wi-Fi devices ?

(a) Bluetooth uses 2.4 GHz radio frequency band, whereas Wi-Fi can use 2.4 GHz or 5 GHz frequency band.

(b) Bluetooth is used for wireless local area networks (WLAN) only, whereas Wi-Fi is used for wireless wide area networks (WWAN) only.

(c) When information is transmitted between two devices using blue-tooth technology , the devices have to be in the line of sight of each other, but when Wi-Fi technology is used the devices need not be in the line of sight of each other.

(d) The statements (a) and (b) given above are correct in this context.

Ans: (a)

Solution: (a) Bluetooth uses 2.4 GHz radio frequency band, whereas Wi-Fi can use 2.4 GHz or 5 GHz frequency band is the difference between Bluetooth and Wi-Fi devices. Bluetooth and Wi-Fi are both wireless communication technologies, but they operate on different frequency bands. Bluetooth uses the 2.4 GHz frequency band, while Wi-Fi can operate on both the 2.4 GHz and 5 GHz frequency bands.

(b) The statement “Bluetooth is used for wireless local area networks (WLAN) only, whereas Wi-Fi is used for wireless wide area networks (WWAN) only” is incorrect. Bluetooth and Wi-Fi are both used for wireless local area networks (WLAN), which are networks that cover a limited area such as a home, office, or campus.

(c) When information is transmitted between two devices using Bluetooth technology, the devices do not have to be in the line of sight of each other, but the range is limited to about 10 meters. On the other hand, Wi-Fi technology has a greater range and can transmit data over longer distances without requiring line-of-sight communication.

Therefore, the correct answer is (a) – Bluetooth uses 2.4 GHz radio frequency band, whereas Wi-Fi can use 2.4 GHz or 5 GHz frequency band.

43. With reference to micro-irrigation, which of the following statements is/are correct ?

1. Fertilizer/nutrient loss can be reduced

2. It is the only means of irrigation in dry land farming

3. In some areas of farming, receding of ground water table can be checked

Select the correct answer using the codes given below :

(a) 1 only

(b) 2 and 3 only

(c) 1 and 3 only

(d) 1,2 and 3

Ans: (c)

Solution: S11 and S3 only are correct statements with reference to micro-irrigation.

  1. Fertilizer/nutrient loss can be reduced through micro-irrigation, as the water and nutrients are directly applied to the root zone of the plants, minimizing runoff and leaching.
  2. Micro-irrigation is not the only means of irrigation in dry land farming. Other methods such as rainwater harvesting, drip irrigation, and sprinkler irrigation can also be used.
  3. In some areas of farming, receding of ground water table can be checked through micro-irrigation, as the water is applied directly to the root zone of the plants, reducing wastage and allowing for more efficient use of water resources.

Therefore, the correct answer is (c) 1 and 3 only.

44. With reference to the period of colonial rule in India, “Home Charges “ formed an important part of drain of wealth from India. Which of the following funds constituted “Home Charges’’ ?

1. Funds used to support the India office in London.

2. Funds used to pay salaries and pensions of British personnel engaged in India.

3. Funds used for waging wars outside India by the British.

(a) 1 only

(b) 1 and 2 only

(c) 2 and 3 only

(d) 1,2,and 3

Ans: (b)

Solution: S1 and S2 only constituted the “Home Charges” during the colonial rule in India.

  1. Funds used to support the India office in London, which was responsible for the administration of India and managing the affairs of the British in India.
  2. Funds used to pay salaries and pensions of British personnel engaged in India, including the salaries of the British civil servants, army personnel, and police force.
  3. Funds used for waging wars outside India by the British were not considered part of the “Home Charges.”

Therefore, the correct answer is (b) 1 and 2 only.

45. What was the reason for mahatma Gandhi to organize a satyagraha on behalf of the peasants off Kheda ?

1. The administration did not suspend the land revenue collection in spite of a drought.

2. The administration proposed to introduce permanent settlement in Gujarat.

Which of the statements given above is/are correct ?

(a) 1 only

(b) 2 only

(c) Both 1 and 2

(d) Neither 1 nor 2

Ans: (a)

Solution: Mahatma Gandhi organized a satyagraha on behalf of the peasants of Kheda in Gujarat in 1918. The reason for this satyagraha was that the administration did not suspend the land revenue collection in spite of a severe drought that had affected the region. The peasants were unable to pay the revenue, and their crops had failed due to the drought. Gandhi argued that the administration should show compassion and suspend the collection of revenue until the drought was over, and the peasants were in a position to pay.

The administration, however, refused to suspend the collection of revenue, and this led to the satyagraha. The satyagraha was successful, and the administration eventually agreed to suspend the collection of revenue until the drought was over.

Therefore, the correct answer is (a) 1 only.

46. The 2004 Tsunami made people realize that mangroves can serve as a reliable safety hedge against coastal calamities. How do mangroves function as a safety hedge ?

(a) The mangrove swamps separate the human settlements from the sea by a wide zone in which people neither live nor venture out.

(b) The mangroves provide both food and medicines which people are in need of after any natural disaster.

(c) The mangrove trees are tall with dense canopies and serve as an excellent shelter during a cyclone or Tsunami.

(d) The mangrove trees do not get uprooted by storms and tides because of their extensive roots.

Ans: (d)

Solution: Mangrove trees have extensive root systems that can reach up to 3 meters deep into the soil, making them highly resistant to strong winds, storms, and tides. During a Tsunami, the mangroves can help reduce the impact of the waves by dissipating the energy and slowing down the flow of water. They also act as a buffer zone, absorbing the force of the waves before they hit the coastline. Mangroves can, therefore, serve as a reliable safety hedge against coastal calamities like Tsunamis and cyclones.

47. The Jain philosophy holds that the world is created and maintained by ?

(a) Universal law

(b) Universal truth

(c) Universal faith

(d) Universal soul

Ans: (a)

Solution: The Jain philosophy holds that the world is created and maintained by the Universal law.

48. Salinization occurs when the irrigation water accumulated in the soil evaporates, leaving behind salts and minerals. What are the effects of Salinization on the irrigated land ?

(a) It greatly increases the crop production.

(b) It makes some soils impermeable .

(c) It raises the water table.

(d) It fills the air spaces in the soil with water.

Ans: (b)

Solution: The accumulation of salts and minerals in the soil can lead to the formation of a crust on the surface, which can make the soil impermeable. This can prevent water from penetrating the soil, which can lead to poor crop growth and reduced crop yields.

49. The “Red Books’’ published by the international union for conservation of nature and natural resources (IUCN) contain lists of ?

1. Endemic plant and animal species present in the biodiversity hotspots.

2. Threatened plant and animal species.

3. Protected sites for conservation of nature and natural resources in various countries.

Select the correct answer using the codes given below :

(a) 1 and 3

(b) 2 only

(c) 2 and 3

(d) 3 only

Ans: (b)

Solution: The Red Data Book is a record of endangered and threatened species of plants, animals, and fungi. It was first created by the International Union for Conservation of Nature (IUCN) in 1964 and has since become a widely recognized tool for identifying and documenting species that are at risk of extinction.

The Red Data Book is an important conservation tool that helps to raise awareness about the threats to biodiversity and provides a framework for conservation efforts. It contains detailed information about the species, including their distribution, habitat, threats, and conservation status. It also includes information about the measures being taken to protect the species and their habitat, as well as recommendations for further action.

50. Why is the offering of “teaser loans’’ by commercial banks a cause of economic concern ?

1. The teaser loans are considered to be an aspect of sub-prime lending and banks may be exposed to the risk of defaulters in future.

2. In India, the teaser loans are mostly given to inexperienced entrepreneurs to set up manufacturing or export units.

Which of the statements given above is/are correct?

(a) 1 only

(b) 2 only

(c) Both 1 and 2

(d) Neither 1 nor 2

Ans: (c)

Solution: A teaser loan, also known as an introductory rate loan, is a type of loan that offers an initial low interest rate for a limited period, usually for the first few months or years. After the introductory period, the interest rate on the loan adjusts to a higher rate, often based on an index such as the prime rate.

Teaser loans are commonly used for mortgages and credit cards, and they can be attractive to borrowers who want to take advantage of the low initial rate to make payments more affordable in the short term. However, it is important to carefully review the loan terms, including the duration of the introductory period and the rate adjustment mechanism, to ensure that the loan is a good fit for your long-term financial goals and budget.

51. An artificial satellite orbiting around the earth does not fall down. This is so because the attraction of earth ?

(a) Does not exist at such distance

(b) Is neutralized by the attraction of the moon

(c) Provides the necessary speed for its steady motion

(d) Provides the necessary acceleration for its motion

Ans: (d)

Solution: An artificial satellite in orbit around the Earth is constantly falling towards the Earth, but it is also moving horizontally at a sufficient speed that it continuously misses the Earth. This balance between the gravitational pull of the Earth and the forward motion of the satellite is what keeps the satellite in orbit without falling back to Earth.

Therefore, the attraction of the Earth is crucial for keeping the satellite in orbit by providing the necessary acceleration for its motion. The attraction of the Moon and the distance between the Earth and the satellite do not play a significant role in this scenario.

52. In the context of Indian economy, consider the following statements ?

1. The growth rate of GDP has steadily increased in the last five years.

2. The growth rate in per capita income has steadily increased in the last five years.

Which of the statements given above is/are correct ?

(a) 1 only

(b) 2 only

(c) Both 1 and 2

(d) Neither 1 nor 2

Ans: (d)

Solution: Fact Based Question

53. In India, which of the following have the highest share in the disbursement of credit to agriculture and allied activities ?

(a) Commercial banks

(b) Cooperative banks

(c) Regional rural banks

(d) Microfinance institutions

Ans: (a)

Solution: According to the latest available data, commercial banks have the highest share in the disbursement of credit to agriculture and allied activities in India, followed by cooperative banks, regional rural banks, and then microfinance institutions.

As per the Reserve Bank of India (RBI), in the financial year 2019-20, commercial banks disbursed about 52% of the total credit to agriculture and allied activities, cooperative banks accounted for 23%, regional rural banks provided 18%, and microfinance institutions gave 7% of the total credit.

Therefore, the correct answer is (a) Commercial banks.

54. Which of the following can aid in furthering the government’s objective of inclusive growth ?

1. Promoting self-help groups.

2. Promoting micro. Small and medium enterprises.

3. Implementing the right to education act.

Select the correct answer using the codes given below :

(a) 1 only

(b) 1 and 2 only

(c) 2 and 3 only

(d) 1,2 and 3 only

Ans: (d)

Solution: The concept of inclusive growth refers to a process of growth that is broad-based, participatory, and sustainable. It aims to reduce poverty and promote shared prosperity by ensuring that the benefits of economic growth are widely distributed across all segments of society, particularly the poor and marginalized. The following options can aid in furthering the government’s objective of inclusive growth:

  1. Promoting self-help groups: Self-help groups (SHGs) are community-based organizations that help people, particularly women, to come together, save their money, and obtain access to credit and other financial services. By promoting SHGs, the government can empower women, provide them with a source of income, and improve their social and economic status.
  2. Promoting micro, small, and medium enterprises: Micro, Small, and Medium Enterprises (MSMEs) are a significant source of employment and income in the economy, particularly in rural areas. By promoting MSMEs, the government can create employment opportunities, boost entrepreneurship, and promote inclusive economic growth.
  3. Implementing the right to education act: The Right to Education Act (RTE) aims to provide free and compulsory education to all children between the ages of 6 and 14. By implementing the RTE, the government can improve access to education, particularly for children from disadvantaged communities, and promote human capital development.

55. Why is the government of India disinvesting its equity in the central public sector enterprises (CPSEs) ?

1. The government intends to use the revenue earned from the disinvest-ment mainly to pay back the external debt.

2. The government no longer intends to retain the management control of the CPSEs.

Which the correct statements given above is/are correct ?

(a) 1 only

(b) 2 only

(c) Both 1 and 2

(d) Neither 1 nor 2

Ans: (d)

Solution: Disinvestment refers to the process of selling or diluting the stake or ownership of the government in public sector enterprises or other entities.

The primary objective of disinvestment is to raise funds for the government, reduce the fiscal burden, improve the operational efficiency of the enterprises, and promote private sector participation in the economy. Disinvestment can also lead to better management practices, increased competition, and a more dynamic market environment.

56. What is difference between asteroids and comets ?

1. Asteroids are small rocky planetoids, while comets are formed of frozen gases held together by rocky and metallic material.

2. Asteroids are found mostly between the orbits of Jupiter and mars, while comets are found mostly between Venus and mercury.

3. Comets show a perceptible glowing tail, while asteroids do not.

Which of the statements given above is/are correct ?

(a) 1 and 2 only

(b) 1 and 3 only

(c) 3 only

(d) 1,2 and 3

Ans: (b)

Solution: Asteroids and comets are both celestial bodies in our solar system but have different characteristics. The differences between asteroids and comets are as follows:

  1. Composition: Asteroids are small, rocky or metallic objects that are left over from the formation of the solar system. Comets, on the other hand, are made up of frozen gases, such as water, methane, and ammonia, as well as rocky and metallic material.
  2. Location: Asteroids are mostly found in the asteroid belt between the orbits of Mars and Jupiter. Comets, on the other hand, are found in various regions of the solar system but are most commonly found in the Kuiper Belt beyond the orbit of Neptune and the Oort Cloud that surrounds the solar system.
  3. Appearance: Comets have a glowing tail that can be seen from Earth when they come closer to the sun. The tail is formed due to the sublimation of the frozen gases as the comet gets closer to the sun. Asteroids do not have a glowing tail and are mostly dull in appearance.

57. Economic growth is usually coupled with ?

(a) Deflation

(b) Inflation

(c) Stagflation

(d) Hyperinflation

Ans: (b)

Solution: Economic growth refers to the increase in the production of goods and services in an economy over a period of time. It is often accompanied by inflation, which refers to the sustained increase in the general price level of goods and services in an economy. There are several reasons why economic growth is often coupled with inflation, including:

  1. Increased demand: Economic growth usually leads to an increase in demand for goods and services, which puts upward pressure on prices.
  2. Increased production costs: As firms increase production to meet the growing demand, their costs of production, such as labor and raw materials, may increase, which can also lead to higher prices.
  3. Increased money supply: Central banks often increase the money supply to support economic growth, which can lead to an increase in spending and inflation.

58. The lowering of bank rate by the reserve bank of India leads to ?

(a) More liquidity in the market

(b) Less liquidity in the market

(c) No change in the liquidity in the market

(d) Mobilization of more deposits by commercial banks

Ans: (a)

Solution: When the Reserve Bank of India (RBI) lowers the bank rate, it reduces the cost at which banks can borrow money from the RBI. This reduction in the cost of borrowing encourages banks to borrow more from the RBI and lend more to businesses and individuals. As a result, there is more money available in the market, which increases liquidity.

The increase in liquidity in the market can lead to several outcomes, including:

  1. Lower interest rates: As there is more money available in the market, interest rates tend to go down.
  2. Increased investment: With lower interest rates, businesses and individuals are more likely to invest in projects, which can lead to economic growth.
  3. Increased consumption: With more money available, consumers may increase their spending, which can also stimulate economic growth.

59. Westerlies in southern hemisphere are stronger and persistent than in northern hemisphere. Why ?

1. Southern hemisphere has less landmass as compared to northern hemisphere.

2. Coriolis force is higher in southern hemisphere as compared to northern hemisphere.

Which of the statements given above is/are correct ?

(a) 1 only

(b) 2 only

(c) Both 1 and 2

(d) Neither 1 nor 2

Ans: (a)

Solution: The Coriolis force is an inertial force that appears to act on objects that are in motion relative to a rotating reference frame, such as the Earth. It is named after French mathematician and engineer Gaspard-Gustave de Coriolis, who first described the phenomenon in the 19th century.

The Coriolis force arises due to the rotation of the Earth, which causes objects in motion to be deflected to the right in the northern hemisphere and to the left in the southern hemisphere. This deflection is a result of the combination of the object’s motion and the Earth’s rotation.

The strength of the Coriolis force depends on the speed of the object, the direction of its motion, and its distance from the equator. The force is strongest at the poles and weakest at the equator. The Coriolis force plays a significant role in the behavior of large-scale weather systems, ocean currents, and the rotation of the Earth.

S1: As the southern hemisphere has less landmass as compared to the northern hemisphere, hence westerlies stronger in the southern hemisphere. S1 is correct. 

S2: The Coriolis force is not stronger in the southern hemisphere than that of northern hemisphere.  S2 is incorrect. 

60. Between India and east Asia, the navigation-time and distance can be greatly reduced by which of the following ?

1. Deepening the Malacca straits between Malaysia and Indonesia.

2. Opening a new canal across the kra isthmus between the gulf of Siam and Andaman sea.

Which of the statements given above is/are correct ?

(a) 1 only

(b) 2 only

(c) Both 1 and 2

(d) Neither 1 nor 2

Ans: (b)

Solution: India and East Asia are currently connected by sea routes that pass through the Strait of Malacca, which is a narrow waterway between Indonesia and Malaysia. While the Strait of Malacca is the most direct route between India and East Asia, it is also a congested and potentially hazardous waterway, with a high risk of piracy and other security threats.

Malacca Strait
Malacca Strait

Opening a new canal across the Kra Isthmus, which is a narrow strip of land that separates the Gulf of Thailand from the Andaman Sea, would provide an alternative route that would bypass the Strait of Malacca and reduce navigation time and distance between India and East Asia.

Statement 2 is correct, as opening a new canal across the Kra Isthmus would indeed reduce navigation time and distance between India and East Asia.

Kra Isthmus
Kra Isthmus

Statement 1, however, is incorrect, as deepening the Strait of Malacca would not significantly reduce navigation time or distance. Therefore, the correct answer is (b) 2 only.

61. Which one of the following is not a feature of “value added tax” ?

(a) It is multi-point destination-based system of taxation.

(b) It is a tax levied on value addition at each stage of transaction in the production-distribution chain.

(c) It is a tax on the final consumption of goods or services and must ultimately be borne by the consumer.

(d) It is basically a subject of the central government and the state governments are only a facilitator for its successful implementation.

Ans: (d)

Solution: Value Added Tax (VAT) is a tax levied on the value added at each stage of production, distribution, and sale of goods and services. It is a multi-point destination-based system of taxation, which means that it is levied on each stage of value addition, from production to the final consumption. The ultimate burden of VAT falls on the final consumer, as it is a tax on the final consumption of goods or services. The VAT system is implemented by both the central and state governments, with the central government collecting taxes on goods that are produced and the state governments collecting taxes on goods that are sold within the state.

62. A “closed economy’’ is an economy in which ?

(a) The money supply is fully controlled

(b) Deficit financing takes place

(c) Only exports take place

(d) Neither exports nor imports take place

Ans: (d)

Solution: A “closed economy” is an economy in which neither exports nor imports take place. It is a self-sufficient economy that does not engage in trade with other countries. All economic activities, such as production, consumption, and investment, take place within the boundaries of the closed economy.

In a closed economy, the money supply can be fully controlled by the central bank or the government as there is no inflow or outflow of foreign currencies. Deficit financing can also take place in a closed economy if the government decides to spend more than it collects in taxes. However, this is not a defining feature of a closed economy.

Although no country in the global economy can be a fully closed economy however, North Korea to an extent is an example of closed economy.

63. When the bark of a tree is removed in a circular fashion all around near its base, it gradually dries up and dies because ?

(a) Water from soil cannot rise to aerial parts

(b) Roots are starved of energy

(c) Tree is infected by soil microbes

(d) Roots do not receive oxygen for respiration

Ans: (b)

Solution: When the bark of a tree is removed in a circular fashion all around near its base, a process known as girdling, it gradually dies because the energy formed by the process of photosynthesis can not reach the roots and roots will starve of energy. Note that Xylem is located in the central part and it will not get affected by removal of peripheral barks but the Phloem will be disturbed as it is located on the periphery.

Characteristic Xylem Phloem
Function Transports water and minerals from roots to aerial parts of the plant Transports sugars, amino acids, and other organic compounds from leaves to the rest of the plant
Composition Made up of tracheids, vessels, fibers, and parenchyma cells Made up of sieve tubes, companion cells, fibers, and parenchyma cells
Location Located in the center of the stem Located on the outer part of the stem, just under the bark
Direction of flow Flows from roots to aerial parts of the plant Flows from leaves to the rest of the plant
Movement of substances Moves passively through the process of transpiration Moves actively through the process of translocation
Structure Lacks living protoplasts when mature Contains living protoplasts
Types Two types: protoxylem and metaxylem Two types: sieve tubes and companion cells

64. The “New START” treaty was in the news. What is this treaty ?

(a) It is a bilateral strategic nuclear arms reduction treaty between the USA and the Russian federation.

(b) It is a multilateral energy security cooperation treaty among the members of the east Asia summit.

(c) It is a treaty between the Russian federation and the European union for the energy security cooperation.

(d) It is a multilateral cooperation treaty among the BRICS countries for the promotion of trade.

Ans: (a)

Solution: The “New START” treaty is a bilateral strategic nuclear arms reduction treaty between the United States and the Russian Federation. It was signed in 2010 and entered into force on February 5, 2011. The treaty limits the number of deployed strategic nuclear warheads to 1,550 for each country and also sets limits on the number of deployed intercontinental ballistic missiles (ICBMs), submarine-launched ballistic missiles (SLBMs), and heavy bombers. The treaty was set to expire in 2021, but the United States and Russia agreed to extend it for another five years until 2026.

65. Three of the following criteria have contributed to the recognition of western Ghats , Sri Lanka and Indo-Burma regions as hotspots of biodiversity :

1. Species richness.

2. Vegetation density.

3. Endemism.

4. Ethno-botanical importance.

5. Threat perception.

6. Adaption of flora and fauna to warm and humid conditions.

Which three of the above are correct criteria in this context ?

(a) 1,2, and 6

(b) 2,4 and 6

(c) 1,3 and 5

(d) 3,4 and 6

Ans: (c)

Solution: The correct criteria for the recognition of Western Ghats, Sri Lanka, and Indo-Burma regions as hotspots of biodiversity are:

  1. Species richness
  2. Endemism
  3. Threat perception

Vegetation density, ethno-botanical importance, and adaptation of flora and fauna to warm and humid conditions are also important factors, but they are not among the main criteria used for identifying biodiversity hotspots.

66. Human activities in the recent past have caused the increased concentration of carbon dioxide in the atmosphere, but a lot of it does not remain in the lower atmosphere because of ?

1. Its escape into the outer stratosphere.

2. The photosynthesis by phytoplankton in the oceans.

3. The trapping of air in the polar ice caps.

Which of the statements given above is/are correct ?

(a) 1 and 2

(b) 2 only

(c) 2 and 3

(d) 3 only

Ans: (b)

Solution: Human activities in the recent past have indeed caused the increased concentration of carbon dioxide in the atmosphere. While some of it is absorbed by plants and oceans through photosynthesis, the majority of it remains in the lower atmosphere and contributes to global warming. Hence, S2 is correct. 

Carbon dioxide does not escape into the outer stratosphere because it is a heavy gas and is not lifted to higher altitudes. So, S1 i incorrect. 

Similarly, the trapping of air in the polar ice caps does not much affect the concentration of carbon dioxide in the atmosphere. S3 is incorrect. 

67. In the context of ecosystem productivity, marine upwelling zones are important as they increase the marine productivity by bringing the ?

1. Decomposer microorganisms to the surface.

2. Nutrients to the surface.

3. Bottom-dwelling organisms to the surface.

Which of the statements given above is/are correct ?

(a) 1 and 2

(b) 2 only

(c) 2 and 3

(d) 3 only

Ans: (b)

Solution: Marine upwelling zones are areas where deep, nutrient-rich water rises to the surface, replacing the warmer surface water. This brings nutrients such as nitrates, phosphates, and silicates to the surface, which promotes the growth of phytoplankton, the base of the marine food chain. The increased productivity in these areas can support large populations of fish and other marine organisms. Decomposer microorganisms and bottom-dwelling organisms are not directly brought to the surface by upwelling.

68. If a tropical rain forest is removed, it does not regenerate quickly as compared to a tropical deciduous forest. This is because ?

(a) The soil of rain forest is deficient in nutrients.

(b) Propagules of the trees in a rain forest have poor viability.

(c) The rain forest species are slow-growing.

(d) Exotic species invade the fertile soil of rain forest.

Ans: (a)

Solution: Tropical rain forests have a very complex and delicate ecosystem. The soil in these forests is generally nutrient-poor and the majority of the nutrients are tied up in the living biomass of the forest. When the forest is cleared, the nutrients are quickly leached away by heavy rain, leaving the soil barren and infertile. Additionally, the high temperatures and high humidity in tropical rain forests lead to rapid decomposition of organic matter, which further depletes the soil of nutrients.

In contrast, tropical deciduous forests have more fertile soils and a greater abundance of nutrients. They also have a higher diversity of species with faster growth rates and better adaptation to disturbance, allowing for quicker regeneration.

69. The Himalayan Range is very rich in species diversity. Which one among the following is the most appropriate reason for this phenomenon ?

(a) It has a high rainfall that supports luxuriant vegetative growth.

(b) It is a confluence of different bio-geographical zones.

(c) Exotic and invasive species have not been invasive species have not been introduced in this region.

(d) It has less human interference.

Ans: (b)

Solution: The Himalayan Range (especially Eastern Part) is one of the world’s biodiversity hotspots and is known for its high species diversity. This is primarily due to its unique geographic location at the confluence of several different bio-geographical zones, including the Palearctic, Indo-Malayan, and the Tibetan Plateau. The range encompasses a wide range of elevations, from tropical forests to alpine meadows, providing a variety of habitats for a diverse array of species.

While high rainfall, low human interference, and the absence of invasive species may also contribute to the richness of the Himalayan ecosystem, the most significant factor is the convergence of different bio-geographical zones, which has led to a high degree of species diversity and endemism.

70. With reference to India, consider the following central acts :

1. Import and export (control) act, 1947

2. Mining, and mineral development (regulation) act, 1957

3. Customs act, 1962

4. Indian forest act, 1927

Which of the above acts have relevance to/bearing on the biodiversity conservation in the country ?

(a) 1 and 3 only

(b) 2,3 and 4 only

(c) 1,2,3 and 4

(d) None of the above acts

Ans: (c)

Solution: Out of the given central acts, the Mining and Mineral Development (Regulation) Act, 1957, has relevance to/bearing on biodiversity conservation as it provides for the regulation of mines and the development of minerals in order to protect the environment and conserve biodiversity.

The Customs Act, 1962, has relevance as it regulates the import and export of goods, including wildlife and biodiversity-related products, to prevent their illegal trade and protect biodiversity.

The Indian Forest Act, 1927, is also relevant to biodiversity conservation as it provides for the protection and management of forests and wildlife in India. It regulates the diversion of forest land for non-forestry purposes, prevents unauthorized occupation of forest land, and empowers forest officials to take necessary measures to protect forests and wildlife.

The Import and Export (Control) Act, 1947, primarily deals with the regulation of trade and commerce and does have an indirect bearing on biodiversity conservation.

71. Karl Marx explained the process of class struggle with the help of which one of the following theories ?

(a) Empirical liberalism

(b) Existentialism

(c) Darwin’s theory of evolution

(d) Dialectical materialism

Ans: (d)

Solution: Dialectical materialism is a philosophical approach to understanding reality that emphasizes the role of material conditions in shaping human history and social relations. According to Marx, the class struggle between the proletariat (working class) and the bourgeoisie (capitalist class) was the driving force of history, and this struggle would ultimately lead to the overthrow of the bourgeoisie and the establishment of a communist society.

72. A layer in the earth’s atmosphere called ionosphere facilities radio communication. Why ?

1. The presence of ozone causes the reflection of radio waves to earth.

2. Radio waves have a very long wavelength.

Which of the statements given above is/are correct ?

(a) 1 only

(b) 2 only

(c) Both 1 and 2

(d) Neither 1 nor 2

Ans: (d)

Solution: The Ionosphere is a layer in the Earth’s atmosphere that contains a high concentration of ions and free electrons, which makes it capable of reflecting radio waves back to the Earth’s surface. This is because radio waves can be refracted or bent by the charged particles in the ionosphere, allowing them to follow the curvature of the Earth and travel long distances without being absorbed by the atmosphere.

The presence of ozone in the Earth’s atmosphere is important for protecting us from harmful ultraviolet radiation from the sun, but it does not play a direct role in the reflection of radio waves in the ionosphere. Similarly, while radio waves do have long wavelengths compared to other types of electromagnetic radiation like visible light, this fact alone does not explain why they are able to be reflected by the ionosphere.

73. Both foreign direct investment (FDI) and foreign institutional investor (FII) are related to investment in a country. Which one of the following statements best represents an important difference between the two ?

(a) FII helps bring better management skills and technology. While FDM only brings in capital.

(b) FII helps in increasing capital availability in general, while FDI only targets specific.

(c) FDI flows only into the secondary market, in general, while FDI only targets specific sectors.

(d) FII is considered to be more stable than FDI.

Ans: (b)

Solution: Foreign Direct Investment (FDI) refers to an investment made by a foreign company or individual in a country that involves establishing business operations or acquiring a stake in an existing business. FDI is typically a long-term investment that involves the transfer of capital, technology, and management skills from the foreign company to the domestic company.

Foreign Institutional Investment (FII), on the other hand, refers to the investment made by foreign institutions, such as mutual funds, pension funds, and hedge funds, in the financial markets of a country. FII is generally a short-term investment and does not involve the transfer of technology or management skills.

Criteria Foreign Direct Investment (FDI) Foreign Institutional Investment (FII)
Nature of Investment Involves establishing operations or acquiring a stake in an existing business in the host country Investment in financial markets of a country
Time Horizon Long-term investment Short-term investment
Control FDI allows the foreign investor to have significant control over the business operations and management of the invested company FII does not involve management or control over the company
Technology Transfer Involves transfer of technology, knowledge, and management practices from the foreign company to the domestic company Does not involve transfer of technology or management practices
Investment Size Generally involves a large investment Generally involves a smaller investment compared to FDI
Risk FDI is considered to be riskier due to the significant investment and involvement in the management of the company FII is considered to be less risky compared to FDI due to its short-term nature
Impact on Economy FDI is believed to have a positive impact on the economy as it creates jobs, transfers technology, and stimulates growth FII can create volatility in the stock market and can lead to short-term capital inflows and outflows, which can impact the economy.

74. A genetically engineered form of Brinjal, known as the Bt-Brinjal, has been developed. The objective of this is ?

(a) To make it pest-resistant

(b) To improve its taste and nutritive qualities

(c) To make it drought-resistant

(d) To make its shelf-life longer

Ans: (a)

Solution: Bt-Brinjal is a genetically modified variety of brinjal that has been developed to be resistant to the fruit and shoot borer (FSB), which is a common pest that can cause significant damage to the plant. The Bt-Brinjal variety has been created by inserting a gene from the bacterium Bacillus thuringiensis (Bt) into the plant’s DNA, which allows it to produce a protein toxic to the FSB.

75. With reference to “Aam Admi Bima yojna’’ consider the following statements ?

1. The member insured under the scheme must be the head of the family or an earning member of the family in a rural landless house-hold.

2. The member insured must be in the age group of 30 to 65 years.

3. There is a provision for free scholarship insured who are studying between classes 9 and 12.

Which of the statements given above is/are correct ?

(a) 1 only.

(b) 2 and 3 only.

(c) 1 and 3 only.

(d) 1,2 and 3.

Ans: (c)

Solution: The correct statements regarding the “Aam Admi Bima Yojana” are:

The member insured under the scheme must be the head of the family or an earning member of the family in a rural landless household. – Correct
The member insured must be in the age group of 18 to 59 years. – This statement is incorrect. The correct age group is 18 to 70 years.
There is a provision for a free scholarship for insured who are studying between classes 9 and 12. – Correct

76. Regular intake of fresh fruits and vegetables is recommended in the diet since they are a good source of antioxidants. How do antioxidants help a person maintain health and promote longevity ?

(a) They activate the enzymes necessary for vitamin synthesis in the body and help prevent vitamin deficiency.

(b) They prevent excessive oxidation of carbohydrates, fats and proteins in the body and help avoid unnecessary wastage of energy.

(c) They neutralize the free radicals produced in the body during metabolism.

(d) They activate certain genes in the cells of the body and help delay the ageing process.

Ans: (c)

Solution: Antioxidants are compounds that can neutralize free radicals, which are unstable molecules produced during normal metabolism or in response to environmental factors like pollution and radiation. Free radicals can damage cells and contribute to the development of various diseases, including cancer, heart disease, and Alzheimer’s disease. By neutralizing free radicals, antioxidants help protect cells from damage and maintain overall health.

There is some evidence to suggest that a diet rich in antioxidants may help promote longevity and delay the aging process, but the exact mechanisms by which this occurs are not fully understood. It is not correct to say that antioxidants activate enzymes for vitamin synthesis or prevent excessive oxidation of carbohydrates, fats, and proteins in the body. While vitamins and energy metabolism are important for health, they are not directly related to the role of antioxidants in the body.

77. Regarding the Indus valley civilization , consider the following statements ?

1. It was predominantly a secular civilization and the religious element, though present, did not dominate the scene.

2. During this period, cotton was used for manufacturing textiles in India.

Which of the statements given above is/are correct ?

(a) 1 only

(b) 2 only

(c) Both 1 and 2

(d) Neither 1 nor 2

Ans: (c)

Solution: Both statements are correct.

  1. The Indus Valley Civilization was predominantly a secular civilization, with evidence of trade, urban planning, and technological advancements indicating a focus on material prosperity rather than religious or spiritual pursuits. While religious symbols and artifacts have been found, they do not dominate the scene.
  2. The use of cotton for manufacturing textiles was one of the significant achievements of the Indus Valley Civilization. Archaeological evidence suggests that cotton was grown, spun, and woven into fabric in the Indus Valley region, with some of the earliest examples of cotton textiles found in the excavations of Mohenjo-Daro and Harappa.

78. The lower Gangetic plain is characterised by humid climate with high temperature throughout the year. Which one among the following pairs of crops is most suitable for this region ?

(a) Paddy and cotton

(b) Wheat and jute

(c) Paddy and jute

(d) Wheat and cotton

Ans: (c)

Solution: The lower Gangetic plain has a humid climate with high temperatures throughout the year, and receives abundant rainfall. These conditions are ideal for the cultivation of paddy (rice) and jute.

Paddy requires a lot of water and is usually grown in flooded fields. The high humidity and abundant rainfall in the lower Gangetic plain make it an ideal region for paddy cultivation. Jute, on the other hand, is a cash crop that requires a lot of rainfall and grows well in moist soil. The high humidity and rainfall in the region make it suitable for the cultivation of jute.

Wheat and cotton are not suitable crops for the lower Gangetic plain as they require drier conditions and cannot tolerate high humidity and excessive rainfall.

79. What could be the main reason/reasons for the formation of African and Eurasian desert belt ?

1. It is located in the sub-tropical high pressure cells.

2. It is under the influence of warm ocean currents.

Which of the statements given above is/are correct in this context ?

(a) 1 only

(b) 2 only

(c) Both 1 and 2

(d) Neither 1 nor 2

Ans: (a)

Solution: The African and Eurasian desert belt is located in the sub-tropical high-pressure cells, also known as the horse latitudes. These are regions of high atmospheric pressure and stable atmospheric conditions, which inhibit the formation of clouds and precipitation. The air in these regions sinks, leading to warming and drying of the land surface, and hence the formation of deserts.

The warm ocean currents do not have a significant influence on the formation of deserts. In fact, the presence of cold ocean currents, such as the Benguela Current off the west coast of southern Africa, can contribute to the formation of deserts by creating cool, stable atmospheric conditions that inhibit precipitation.

80. The jet aircrafts fly very easily and smoothly in the lower stratosphere. What could be the appropriate explanation ?

1. There are no clouds or water vapour in the lower stratosphere.

2. There are no vertical winds in the lower stratosphere.

Which of the statements given above is/are correct in this context ?

(a) 1 only

(b) 2 only

(c) Both 1 and 2

(d) Neither 1 nor 2

Ans: (c)

Solution: The lower stratosphere is characterized by a stable atmospheric condition with very little vertical mixing of air, which means that there are no strong updrafts or downdrafts that can cause turbulence and affect the flight of aircraft. This absence of vertical winds is the main reason why jet aircraft can fly smoothly in the lower stratosphere.

While the absence of clouds or water vapor may contribute to the smoothness of the flight, they are not the main reason. Clouds and water vapor can cause turbulence and affect the flight of aircraft, but they are not always present in the lower stratosphere.

81. Consider the following statements :

1. Biodiversity is normally greater in the lower latitudes as compared to the higher latitudes.

2. Along the mountain gradients, biodiversity is normally greater in the lower altitudes as compared to the higher altitudes.

Which of the statements given above is/are correct ?

(a) 1 only

(b) 2 only

(c) Both 1 and 2

(d) Neither 1 nor 2

Ans: (c)

Explanation: Biodiversity is normally greater in the lower latitudes as compared to the higher latitudes. This statement is correct. The lower latitudes are closer to the equator and receive more sunlight and rainfall, which provide favorable conditions for a wide variety of species to thrive. In contrast, the higher latitudes are farther from the equator and receive less sunlight and rainfall, making it difficult for many species to survive.

Along the mountain gradients, biodiversity is normally greater in the lower altitudes as compared to the higher altitudes: This statement is also correct. As we move up the altitude gradient of a mountain, the temperature and precipitation levels change, resulting in different habitats and conditions that may not be suitable for many species. Thus, the lower altitudes typically have higher biodiversity than the higher altitudes.

82. The Brahmaputra, Irrawaddy and Mekong rivers originate in Tibet narrow and parallel mountain ranges in their upper reaches. Of these rivers, Brahmaputra makes a “U” turn in its course to flow into India. This “U” turn is due to ?

(a) Uplift of folded Himalayan series.’

(b) Syntaxial bending of geologically young Himalayas.

(c) Geo-tectonic disturbance in the tertiary folded mountain chains.

(d) Both (a) and (b) above.

Ans: (b)

Solution: The Brahmaputra River originates in Tibet and flows eastward, where it takes a sharp turn towards the south, forming a “U” shape, before entering India. This sharp turn is due to the Syntaxial bending of the geologically young Himalayas. The Himalayas are still undergoing tectonic activity, which has resulted in the bending of the mountain range towards the east. This bend creates a low-pressure zone, which attracts the Brahmaputra River towards it and causes it to change direction.

The Irrawaddy and Mekong rivers also originate in Tibet and flow towards Southeast Asia. However, they do not experience the same sharp turn as the Brahmaputra River, as they are not influenced by the same syntaxial bending of the Himalayas.

83. A state in India has the following characteristics :

1. Its northern part is arid and semi-arid.

2. Its central part produces cotton.

3. Cultivation of cash crops is predominant over food crops.

Which one of the following states has all of the above characteristics ?

(a) Andhra Pradesh.

(b) Gujarat.

(c) Karnataka.

(d) Tamil Nadu.

Ans: (b)

Solution: Gujarat is a state in western India and has all the characteristics mentioned in the question. Its northern part, known as Kutch, is arid and semi-arid, while the central part of the state, including the regions of Saurashtra and South Gujarat, is known for cotton production. The cultivation of cash crops such as groundnut, sesame, and castor is predominant in Gujarat, while the production of food crops such as rice and wheat is comparatively low.

84. What is “virtual private network” ?

(a) It is a private computer network of an organization where the remote users can transmit encrypted information through the server of the organization.

(b) It is a computer network across a public internet the provides users access to their organizations network while maintaining the security of the information transmitted.

(c) It is a computer network in which users can access a shared pool of computing resources through a service provider.

(d) None of the statements (a), (b) and (c) given above is correct description of virtual private network.

Ans: (b)

Solution: It is a computer network across a public internet that provides users access to their organization’s network while maintaining the security of the information transmitted.

A virtual private network (VPN) is a technology that enables a secure connection over a public network such as the internet. It allows users to access a private network remotely and securely as if they were directly connected to the private network. VPNs use encryption and tunneling protocols to protect the confidentiality and integrity of data transmitted over the network.

85. The “dharma” and “Rita” depict a central Idea of ancient vedic civilization of India. In this context, consider the following statements :

1. Dharma was a conception of obligations and of the discharge of one’s duties to oneself and to others.

2. Rita was the fundamental moral law governing the functioning of the universe and all it contained.

Which of the statements given above is/are correct ?

(a) 1 only

(b) 2 only

(c) Both 1 and 2

(d) Neither 1 nor 2

Ans: (c)

Solution: Dharma and Rita are two central concepts of ancient Vedic civilization in India. Both of these concepts are interconnected and reflect the ethical and moral values of the society.

Dharma is a complex and multi-layered concept in Hinduism. It refers to the inherent nature or essence of a thing, as well as the duty, righteousness, and social order. It is the idea of fulfilling one’s duties and responsibilities, both to oneself and to society. Dharma is a concept of obligations and responsibilities that are necessary for the maintenance of social order and the harmony of the universe.

Rita is the fundamental moral law governing the functioning of the universe and all it contains. It is the cosmic order that governs the universe, including the natural world and human society. It is the principle of order and balance that regulates the functioning of the universe. The concept of Rita emphasizes the importance of maintaining balance and order in the world.

86. In the context of global oil prices, “Brent crude oil” is frequently referred to in the news. What does this term imply ?

1. It is a major classification of crude oil.

2. It is sourced from north sea.

3. It does not contain sulphur.

Which of the statements given above is/are correct ?

(a) 2 only

(b) 1 and 2 only

(c) 1 and 3 only

(d) 1,2, and 3

Ans: (b)

Solution: Brent crude oil is a major classification of crude oil that is sourced from the North Sea. It is a light and sweet crude oil, which means it has a low density and low sulfur content. Brent crude oil is used as a benchmark for global oil prices and is traded on the Intercontinental Exchange (ICE).

S3 is incorrect as Brent crude oil does contain sulfur, although in lower quantities compared to other types of crude oil.

87. The function of heavy water in a nuclear reactor is to ?

(a) Slow down the speed of neutrons.

(b) Increase the speed of neutrons.

(c) Cool down the reactor.

(d) Stop the nuclear reaction.

Ans: (a)

Solution: Heavy water is a form of water that contains a higher proportion of the isotope deuterium (D) or heavy hydrogen. In a nuclear reactor, heavy water is used as a moderator to slow down the speed of neutrons that are released during the fission process. Neutrons are slowed down as they collide with the heavy water molecules, which increases the chances of a successful nuclear reaction.

The slowing down of neutrons is essential for a sustained nuclear reaction, as fast neutrons are less likely to be captured by the fuel and sustain the chain reaction. Heavy water has a high neutron capture cross-section and does not absorb neutrons as readily as regular water or other moderators, making it an efficient moderator for nuclear reactors.

88. In India, if a religious sect/community is given the status of a national minority, what special advantages it is entitled to ?

1. It can establish and administer exclusive educational institutions.

2. The president of India automatically nominates a representative of the community to Lok Sabha.

3. It can derive benefits from the prime minister’s 15-point programme.

Which of the statements give above is/are correct ?

(a) 1 only

(b) 2 and 3 only

(c) 1 and 3 only

(d) 1,2 and 3

Ans: (c)

Solution: The Constitution of India recognizes religious and linguistic minorities and provides for their protection and promotion. The government may grant minority status to a religious or linguistic community if it meets certain criteria, such as being numerically smaller than the rest of the population in a particular state or region.

If a religious sect/community is given the status of a national minority, it is entitled to the following special advantages:

It can establish and administer exclusive educational institutions. This is provided under Article 30 of the Constitution, which grants all minorities the right to establish and administer educational institutions of their choice.

The President of India does not automatically nominate a representative of the community to Lok Sabha. This statement is incorrect.

The community can derive benefits from the Prime Minister’s 15-point program, which aims to improve the socio-economic conditions of minorities. The program includes initiatives such as providing scholarships, setting up coaching centers, and improving access to credit facilities.

89. India is home to lakhs of person with disabilities, what are the benefits available to them under the law ?

1. Free schooling till the age of 18 years in government- run schools

2. Preferential allotment of land for setting up business

3. Ramps in public buildings

Which of the statements given above is/are correct ?

(a) 1 only

(b) 2 and 3 only

(c) 1 and 3 only

(d) 1,2 and 3

Answer: (d) 

Explanation: Persons with disabilities in India are entitled to a number of benefits under various laws, including the Rights of Persons with Disabilities Act, 2016. Some of the benefits available to them include:

Free education: The law provides for free and compulsory education to all children with disabilities till the age of 18 years in government-run schools.

Preferential allotment of land: Persons with disabilities can apply for allotment of land to set up businesses or for residential purposes. They are given preferential treatment in allotment of land, as per the provisions of the law.

Accessibility: Public buildings, including schools, hospitals, and offices, must be made accessible to persons with disabilities. This includes the provision of ramps, handrails, and other facilities to make the building accessible to all.

90. With what purpose is the government of India promoting the concept of “Mega food parks” ?

1. To provide good infrastructure facilities for the food processing industry.

2. To increase the processing of perishable items and reduce wastage.

3. To provide emerging and eco-friendly food processing technologies to entrepreneurs.

Select the correct answer using the codes given below :

(a) 1 only

(b) 1 and 2 only

(c) 2 and 3 only

(d) 1,2 and 3

Answer: (d) 

Explanation: The Government of India is promoting the concept of “Mega Food Parks” with the objective of providing good infrastructure facilities for the food processing industry, increasing the processing of perishable items and reducing wastage, and providing emerging and eco-friendly food processing technologies to entrepreneurs.

The Mega Food Park Scheme aims to establish a “Farm to Market Infrastructure” to link the agricultural production centers with the market through a well-equipped food processing industry. The scheme provides a mechanism to bring together farmers, processors and retailers and links them through a common platform, thereby facilitating direct marketing of produce to consumers.

91. The authorization for the withdrawal of funds from the consolidated fund of India must come from ?

(a) The president of India

(b) The parliament of India

(c) The prime minister of India

(d) The union finance minister

Ans: (b)

Solution: The authorization for the withdrawal of funds from the consolidated fund of India must come from the parliament of India. The consolidated fund of India is created under Article 266 of the Constitution of India and all revenues received by the Government of India, loans raised by the Government by the issue of treasury bills, loans or ways and means advances, and all moneys received by the Government in repayment of loans, are credited to this fund.

Withdrawals from this fund can only be made with the authorization of the parliament of India through the appropriation act passed annually.

92. All revenues received by the union government by way of taxes and other receipts for the conduct of government business are credited to the ?

(a) Contingency fund of India

(b) Public account

(c) Consolidated fund of India

(d) Deposits and advances fund

Ans: (c)

Solution: All revenues received by the union government by way of taxes and other receipts for the conduct of government business are credited to the Consolidated Fund of India. As per Article 266 of the Constitution of India, the Consolidated Fund of India is the most important government account and all revenues received by the Government of India, loans raised by the Government by the issue of treasury bills, loans or ways and means advances, and all moneys received by the Government in repayment of loans, are credited to this fund.

The Contingency Fund of India is set up under Article 267 of the Constitution of India and is used to meet unforeseen expenditure by the Government of India, while the Public Account of India is used to account for flows of funds that do not belong to the Government of India but are temporarily held by it. The Deposits and Advances Fund is maintained for the deposit of various receipts of the government and also for certain payments.

93. Microfinance is the provision of financial services to people of low-income groups. This includes both the con-summers and the self-employed. The service/services rendered under micro-finance is/are :

1. Credit facilities

2. Savings faculties

3. Insurance facilities

4. Fund transfer faculties

Select the correct answer using the codes given the lists ?

(a) 1 only

(b) 1 and 4 only

(c) 2 and 3 only

(d) 1,2 ,3 and 4

Ans: (d)

Solution: Microfinance is the provision of financial services to people of low-income groups, which includes both consumers and the self-employed. The services provided under microfinance typically include credit facilities, savings facilities, and insurance facilities. Fund transfer facilities may also be offered by some microfinance institutions.

94. Southeast Asia has captivated the attention of global community over space and time as a geostrategically significant region. Which among the following is the most convincing explanation for this global perspective ?

(a) It was the hot theatre during the second world war.

(b) Its location between the Asian powers of china and India.

(c) It was the arena of superpower confrontation during the cold war period.

(d) Its location between the pacific and Indian oceans and its pre-eminent maritime character.

Ans: (d)

Solution: Southeast Asia is located between the Pacific and Indian Oceans, which makes it a crucial transit point for shipping and air traffic. Its maritime character has made it an important region for trade and commerce, as well as for military operations and strategic positioning. Additionally, the region is home to several major ports and waterways, including the Strait of Malacca, which is one of the busiest shipping lanes in the world.

Furthermore, Southeast Asia’s location between the two Asian powers of China and India has also contributed to its geopolitical significance. The region serves as a buffer zone between the two countries, and its political stability is crucial to maintaining peace and stability in the wider region. During the Cold War, Southeast Asia was also the arena of superpower confrontation, with the United States and the Soviet Union vying for influence in the region.

Overall, while all of the options have contributed to Southeast Asia’s geostrategic significance, its location between the Pacific and Indian Oceans and its pre-eminent maritime character are the most convincing explanations for the global perspective.

95. A company marketing food products advertises that its items do not contain trans-fats. What does this campaign signify to the customers ?

1. The food products are not made out of hydrogenated oils.

2. The food products are not made out of animal fats/oils.

3. The oils used are not likely to damage the cardiovascular health of the consumers.

Which of the statements given above is/are correct ?

(a) 1 only.

(b) 2 and 3 only.

(c) 1 and 3 only.

(d) 1,2 and 3.

Ans: (c)

Solution: Trans fats are a type of unsaturated fat that can have negative health effects on the body, particularly on cardiovascular health. They are often found in partially hydrogenated oils, which are commonly used in processed foods to improve their shelf life and texture.

Therefore, if a company markets its food products as not containing trans fats, it signifies that their food products are not made out of hydrogenated oils (statement 1) and that the oils used are not likely to damage the cardiovascular health of the consumers (statement 3).

However, the campaign does not necessarily signify that the food products are not made out of animal fats/oils (statement 2), as animal fats/oils may or may not contain trans fats.

96. Among the following who are eligible to benefit from the “mahatma Gandhi national rural employment guarantee act” ?

(a) Adult members of only the scheduled caste and scheduled tribe households

(b) Adult members of below poverty line (BPL) households

(c) Adult members of households of all backward communities

(d) Adult members of any household

Ans: (d)

Solution: The Mahatma Gandhi National Rural Employment Guarantee Act (MGNREGA) is a social welfare scheme that aims to provide employment opportunities to the rural poor. The scheme guarantees a minimum of 100 days of wage employment in a financial year to every household whose adult members volunteer to do unskilled manual work.

The scheme is not restricted to any particular caste, tribe, or community, and any adult member of a household can benefit from it. However, priority is given to households below the poverty line (BPL), and special provisions are made for the employment of women and members of Scheduled Castes (SCs) and Scheduled Tribes (STs).

97. With reference to look east policy of India , consider the following statements ?

1. India wants to establish itself as an important regional player in the east Asian regional player in the east Asian affairs.

2. India wants to plug the vacuum created by the termination of cold war.

3. India wants to restore the historical and cultural ties with its neighbors in southeast and east Asia.

Which of the statements given above is/are correct ?

(a) 1 only

(b) 1 and 3 only

(c) 3 only

(d) 1,2 and 3

Ans: (b)

Solution: India’s Look East Policy was initiated in the 1990s with the aim of promoting economic, strategic and cultural ties with the countries of Southeast Asia, East Asia and Oceania.

Statement 1 is correct as one of the main objectives of the policy is to establish India as an important regional player in East Asian affairs. The policy aims to enhance India’s economic engagement with the region, increase strategic cooperation and improve cultural ties.

Statement 2, however, is incorrect. The Look East Policy was not initiated to “plug the vacuum created by the termination of the Cold War.” Rather, it was introduced as a response to the changing global economic and strategic landscape, and the growing importance of the East Asian region.

Statement 3 is also correct as the policy aims to restore India’s historical and cultural ties with its neighbors in Southeast and East Asia. These ties date back to ancient times, and the policy seeks to leverage them to deepen India’s engagement with the region.

98. When the annual budget is not passed by the lok sabha ?

(a) The budget is modified and presented again.

(b) The budget is referred to the rajya sabha for suggestions.

(c) The union finance minister is asked to resign.

(d) The prime minister submits the resignation of council of ministers.

Ans: (d)

Solution: If the annual budget is not passed by the Lok Sabha, it is considered a failure of the ruling government to maintain the confidence of the house on the issue of financial matters. This amounts to a vote of no-confidence against the government. In such a situation, the Prime Minister may submit the resignation of the Council of Ministers to the President, who may either accept the resignation or ask the government to prove its majority on the floor of the Lok Sabha through a vote of confidence.

99. Under the constitution of India, which one of the following is not a fundamental duty ?

(a) To vote in public elections

(b) To develop the scientific temper

(c) To safeguard public property

(d) To abide by the constitution and respect its ideals

Ans: (a)

Solution: Fundamental duties are a set of moral obligations prescribed by the Indian Constitution that every citizen is expected to fulfill. These duties were added to the Constitution by the 42nd Amendment Act in 1976.

The ten fundamental duties are:

  1. To abide by the Constitution and respect its ideals and institutions.
  2. To cherish and follow the noble ideals that inspired our national struggle for freedom.
  3. To uphold and protect the sovereignty, unity, and integrity of India.
  4. To defend the country and render national service when called upon to do so.
  5. To promote harmony and the spirit of common brotherhood among all the people of India.
  6. To value and preserve the rich heritage of our composite culture.
  7. To protect and improve the natural environment.
  8. To develop the scientific temper, humanism, and the spirit of inquiry and reform.
  9. To safeguard public property and to abjure violence.
  10. To strive towards excellence in all spheres of individual and collective activity.

Therefore, the correct answer is (a) To vote in public elections, which is not a fundamental duty.

100. With reference to the finance commission of India, which of the following statements is correct ?

(a) It encourages the inflow of foreign capital for infrastructure development.

(b) It facilities the proper distributor of finances among the public section undertakings.

(c) It ensures transparency in financial administration.

(d) None of the statements (a), (b) and (c) given above is correct in his context.

Ans: (d)

Solution: The Finance Commission of India is a constitutional body set up by the President of India every five years to make recommendations on the distribution of tax revenues between the Union government and the states and among the states themselves. Its primary role is to ensure a fair and equitable distribution of resources among the different levels of government in India.

The main functions of the Finance Commission are:

  1. To recommend the distribution of net proceeds of taxes between the Centre and the States, and among the States.
  2. To determine the principles that govern the grants-in-aid to the States from the Consolidated Fund of India.
  3. To review the state of finances of the Union and State Governments and suggest measures to improve their financial position.
  4. To recommend measures for augmenting the resources of panchayats and municipalities.

Therefore, none of the statements (a), (b) and (c) given above is correct in this context.

Download question Paper

UPSC Prelims 2011 Answer Key Set D
UPSC Prelims 2011 Answer Key Set D

Important Links

Leave a Comment

Your email address will not be published. Required fields are marked *